At a potluck, Agatha brings four dishes, Bertha brings three dishes, and five other friends bring no dishes but instead money to help pay for the food. If all the dishes are eaten up, and everyone eats the same amount, what fraction of the money should go to Bertha?

Answers

Answer 1

Answer:

3/7

Step-by-step explanation:

Agatha brings four dishes, Bertha brings three dishes. The total number of dishes brought = dishes brought by Agatha + dishes brought by Bertha.

Total dishes = 4 + 3 = 7 dishes

The remaining five friends brought money for the dishes. Therefore the fraction of money going to Bertha is the ratio of dishes brought to Bertha to the total number of dishes multiplied by the money. Therefore:

Fraction of the money should go to Bertha =  dishes brought by Bertha/total dishes

Fraction of the money should go to Bertha = 3/7 × money


Related Questions

How does a globe represent the fact that there are no parallel lines in elliptical geometry? A. The equator is not parallel to any other latitudinal lines. B. The north and south poles are never connected by a geodesic. C. The geodesics connecting the Nortj and South poles never intersect. D. The geodesics connecting the north and south poles intersect at both of the poles.

Answers

Answer:

The correct option is;

D. The geodesics connecting the North as South poles intersect at both of the poles

Step-by-step explanation:

As an analog to the straight line, the geodesic of a curved surface is the shortest path between two points on the curved surface

Whereby the Earth is assumed to be spherical, the geodesics around the Earth would then be closed circles and like the longitude and latitude, will meet at the North and South poles

However given that the Earth is an Oblate ellipsoid, Euclid's parallel postulate is not in effect and the geodesics connecting the North and South Poles Intersect at both poles.

The answer is Option D, option D is correct.

which is a true statement about an exterior angle of a triangle

Answers

Answer:

D

Step-by-step explanation:

The exterior angles are out of the triangle at all times and it adds up to make 180 with anyone of the inside angle of the triangle.

The pair also rests on the same flat/straight line and that makes a pair.

Therefore we can say that it is formed by a linear pair/group with one of the interior/inside angles of the triangle.

So, the correct answer would be D.

The true statement about an exterior angle of a triangle is C; It forms a linear pair with one of the interiior angles of the triangle .

What is the Exterior Angle of a Triangle Property?

An exterior angle of a triangle is equal to the sum of the opposite interior angles.

We know that the exterior angles are out of the triangle at all times and it adds up to make 180 with anyone of the inside angle of the triangle.

The pair also rests on the same straight line and that makes a pairs.

Therefore we can say that it is formed by a linear pair with one of the interior angles of the triangle.

So, the correct answer would be C.

Learn more about exterior angles;

https://brainly.com/question/14164971

#SPJ2

Help me with this question please‼️‼️

Answers

Answer: 1/2  (choice C)

Explanation:

The gradient is the same as the slope

m = slope

m = rise/run

m = (change in y)/(change in x)

m = (y2-y1)/(x2-x1)

m = (6-1)/(6 - (-4))

m = (6-1)/(6+4)

m = 5/10

m = 1/2

In decimal form, this becomes 0.5

A slope of 1/2 means we move up 1 and to the right 2 units each time to generate points along the diagonal line.

A positive slope goes uphill as we move to the right (due to the positive rise value).

Answer:

One way to find the gradient is using the expression :

[tex]m = \frac{y_{2} - y_{1}}{x_{2} - x_{1}} = \frac{y_{B} - y_{A}}{x_{B} - x_{A}}[/tex]

So [tex]m = \frac{6 - 1}{6- (-4)} =\frac{5}{10} =\frac{1}{2}[/tex]

ABCD RECTANGLE α + β = ?

Answers

Answer:

Step-by-step explanation:

I'm going to walk through this analytically, so I will have to assign some variables to angles that are not marked. Pay close attention so you can follow the logic.

The angle at the top left next to and to the left of 40 will be "x", and the one to the right of 40 will be "y". Because that angle is a right angle, then we know that

x + y + 40 = 90 and

x + y = 50.

We also know that, by the Triangle Angle-Sum Theorem, the 2 triangles that contain alpha and beta will add up to equal 360, 180 apiece. So now we have:

x + 90 + α + y + 90 + β = 360.

Let's regroup a bit:

x + y + α + β + 90 + 90 = 360 and

(x + y) + α + β + 180 = 360.

But we know from above that x + y = 50, so

50 + α + β + 180 = 360 and

230 + α + β = 360 and

α + β = 130. There you go!

Answer:

α + β = 130

Step-by-step explanation:

∠ A = ∠ C = 90°

The sum of the 3 angles in a triangle = 180°

vertex angle at D inside the Δ = 180 - (90 + α ) ← Δ on left

vertex angle at D inside the Δ = 180 - (90 + β ) ← Δ on right

∠ ADC = 90° thus

180 - (90 + α) + 180 - (90 + β) + 40 = 90

180 - 90 - α + 180 - 90 - β + 40 = 90, that is

220 - α - β = 90 (add α and β to both sides )

220 = 90 + α + β (subtract 90 from both sides )

130 = α + β

Which of the following is equivalent to -12 + 3)?
12 + 31
– 12– 3
o 12 – 3
0 12 + 3

Answers

Answer:

[tex]12 - 3[/tex]

Step-by-step explanation:

[tex]( - 12 + 3) \\ = 12 - 3 \\ =- 9[/tex]

Hope it is helpful....

Simplify 18-2[x + (x - 5)].
O8-4x
28 - 4x
28 - 2x

Answers

Answer:

28-4x

Step-by-step explanation:

Step 1: Open most inner bracket and simplify

=18-2(x+x-5)

=18-2(2x-5)

Step 2: Expand brackets by multiplying 2 in and simplify

=18-2(2x)-2(-5)

=18-4x+10

=28-4x

Therefore the answer is 28-4x

A man died leaving property
worth 49000 for his three daughters and a son. Find out the share of each in inheritance?

Answers

Answer:

49000

Step-by-step explanation:

since it's the same worth

Answer:

49000

Step-by-step explanation:

since there was the same worth given to all

What is the value of x?

Answers

Answer:

62 is the answer to your question

one quick way to know the answer is to know that the two opposite interior angles has to add up to the outer external angle, so x + 38 = 100

Answer:

B. 62°

Step-by-step explanation:

First, solve the unknown interior angle. Interior angles and exterior angles add up to 180°.

? + 100 = 180

? = 80

The sum of the angles of a triangle is 180°. Form and solve the following equation.

x + 80 + 38 = 180

x + 118 = 180

x = 62

PLS HELP ME ON THIS QUESTION I WILL MARK YOU AS BRAINLIEST IF YOU KNOW THE ANSWER!!

Answers

Answer: will be A

Step-by-step explanation:

The most reasonable answer would be A

Find the probability of drawing 3 Aces at random from a deck of 52 ordinary playing cards if the cards are:_________
A) Replaced
B) Not Replaced

Answers

Answer:

a. With replacement

1/2197

b. Without replacement

1/5,525

Step-by-step explanation:

Okay, here is a probability question.

The key to answering this question is by knowing the number of aces in a deck of cards.

There is 1 ace per suit, so there is a total of 4 aces per deck of cards.

So, mathematically the probability of picking an ace would be;

number of aces/ total number of cards = 4/52 = 1/13

a. Now since the action is with replacement; that means that at any point in time, the total number of cards would always remain 52 even after making our picks.

So the probability of picking three aces with replacement would be;

1/13 * 1/13 * 1/13 = 1/2197

b. Without replacement

what this action means is that after picking a particular card, we do not return the picked card to the deck of cards.

For the first card picked, we will be having a total of 4 aces and 52 total cards.

So the probability of picking an ace would be 4/52 = 1/13

For the second card picked, we shall be left with selecting an ace out of the remaining 3 aces and the total remaining 51 cards

So the probability will be 3/51 = 1/17

For the third and last card to be picked, we shall be left with picking 1 out of the remaining 2 aces cards and out of the 50 cards left in the deck.

So the probability now becomes 2/50 = 1/25

Thus, the combined probability of picking 3 aces cards without replacement from a deck of cards will be;

1/13 * 1/17 * 1/25 = 1/5,525

Using the binomial and the hypergeometric distribution, it is found that the probabilities are:

a) 0.0005 = 0.05%.

b) 0.0002 = 0.02%.

Item a:

With replacement, hence the trials are independent, and the binomial distribution is used.

Binomial probability distribution

[tex]P(X = x) = C_{n,x}.p^{x}.(1-p)^{n-x}[/tex]

[tex]C_{n,x} = \frac{n!}{x!(n-x)!}[/tex]

The parameters are:

x is the number of successes. n is the number of trials. p is the probability of a success on a single trial.

For this problem:

In a deck, there are 52 cards, of which 4 are Aces, hence [tex]p = \frac{4}{52} = 0.0769[/tex]3 cards are drawn, hence [tex]n = 3[/tex].

The probability is P(X = 3), then:

[tex]P(X = x) = C_{n,x}.p^{x}.(1-p)^{n-x}[/tex]

[tex]P(X = 3) = C_{3,3}.(0.0769)^{3}.(0.9231)^{0} = 0.0005[/tex]

0.0005 = 0.05% probability.

Item b:

Without replacement, hence the trials are not independent and the hypergeometric distribution is used.

Hypergeometric distribution:

[tex]P(X = x) = h(x,N,n,k) = \frac{C_{k,x}C_{N-k,n-x}}{C_{N,n}}[/tex]

[tex]C_{n,x} = \frac{n!}{x!(n-x)!}[/tex]

The parameters are:

x is the number of successes. N is the size of the population. n is the size of the sample. k is the total number of desired outcomes.

In this problem:

Deck of 52 cards, hence [tex]N = 52[/tex].4 of the cards are Aces, hence [tex]k = 4[/tex].3 cards are drawn, hence [tex]n = 3[/tex].

The probability is also P(X = 3), hence:

[tex]P(X = x) = h(x,N,n,k) = \frac{C_{k,x}C_{N-k,n-x}}{C_{N,n}}[/tex]

[tex]P(X = 3) = h(3,52,3,4) = \frac{C_{4,3}C_{48,0}}{C_{52,3}} = 0.0002[/tex]

0.0002 = 0.02% probability.

To learn more about the binomial and the hypergeometric distribution, you can take a look at https://brainly.com/question/25783392

If a quadratic equation is written in intercept form, y = (x-3)(x+5), then vertex is at...

Answers

Answer:

The vertex is at (-1, -16).

Step-by-step explanation:

We are given the quadratic equation:

[tex]y = (x-3)(x+5)[/tex]

And we want to find its vertex.

Recall that the x-coordinate of the vertex is also the axis of symmetry. Since a parabola is symmetric about the axis of symmetry, the axis of symmetry is halfway between the two roots.

From the equation, we can see that our two roots are x = 3 and x = -5.

Hence, the axis of symmetry or the x-coordinate of the vertex is:

[tex]\displaystyle x = \frac{(3) + (-5)}{2} = -1[/tex]

To find the y-coordinate of the vertex, evaluate the equation at x = -1:

[tex]\displaystyle \begin{aligned} y(-1) &= ((-1)-3)((-1)+5)\\ &= (-4)(4) \\&= -16\end{aligned}[/tex]

Hence, the vertex is at (-1, -16).

A local trucking company fitted a regression to relate the travel time (days) of its shipments as a function of the distance traveled (miles). The fitted regression is Time = −7.126 + .0214 Distance, based on a sample of 20 shipments. The estimated standard error of the slope is 0.0053. Find the critical value for a right-tailed test to see if the slope is positive, using α = .05.A local trucking company fitted a regression to relate the travel time (days) of its shipments as a function of the distance traveled (miles). The fitted regression is Time = −7.126 + .0214 Distance, based on a sample of 20 shipments. The estimated standard error of the slope is 0.0053. Find the critical value for a right-tailed test to see if the slope is positive, using α = .05.

Answers

Answer:

1.734

Step-by-step explanation:

Given that:

A local trucking company fitted a regression to relate the travel time (days) of its shipments as a function of the distance traveled (miles).

The fitted regression is Time = −7.126 + .0214 Distance

Based on a sample size n = 20

And an Estimated standard error of the slope = 0.0053

the critical value for a right-tailed test to see if the slope is positive, using ∝ = 0.05 can be computed as follows:

Let's determine the degree of freedom df = n - 1

the degree of freedom df = 20 - 2

the degree of freedom df =  18

At the level of significance ∝ = 0.05 and degree of freedom df =  18

For a right tailed test t, the critical value from the t table is :

[tex]t_{0.05, 18} =[/tex] 1.734

The length and width of a book cover are 22.2 centimeters and 12 centimeters respectively. The actual length (and width) can be 0.3 unit less than the measured length (and width) or 0.3 unit greater than the measured length (and width). a. Find the minimum and maximum possible lengths and widths of the book cover. b. Calculate the minimum and maximum possible areas of the book cover.

Answers

Part (a)

The length is supposed to be 22.2 cm, but it could be 0.3 cm less

So 22.2 - 0.3 = 21.9 cm is the smallest value for the length. This is the lower bound of the length.

The upper bound is 22.2 + 0.3 = 22.5 cm as it is the largest the length can get.

-------------

Use this for the width as well

The width is supposed to be 12 cm, but it could be as small as 12-0.3 = 11.7 cm and as large as 12+0.3 = 12.3 cm

-------------

Answers:smallest length = 21.9 cmlargest length = 22.5 cmsmallest width = 11.7 cmlargest width = 12.3 cm

============================================

Part (b)

Use the smallest length and width to get the smallest possible area

smallest area = (smallest width)*(smallest length) = 11.7*21.9 = 256.23

-------------

Repeat the same idea but for the largest length and width to get the largest possible area

largest area = (largest width)*(largest length) = 12.3*22.5 = 276.75

-------------

Answers:smallest area = 256.23 square cmlargest area = 276.75 square cm

Write the polynomial in standard form 8-5x+x^3-2x^4

Answers

Answer:

-2x^4+x^3-5x+8

Step-by-step explanation:

8-5x+x^3-2x^4

Write the polynomial from highest powers of x to lowest

-2x^4+x^3-5x+8

How do I write a verbal expression for each algebraic expression?

23f

Answers

Answer:

Twenty three times f

Step-by-step explanation:

Please explain and help

Answers

Answer:

y=-x+2  

Step-by-step explanation:

it is linear equation y=mx+b two points (0,2),(1,1)

find m ( slope)=y2-y1/x2-x1 ⇒1-2/1-0⇒-1

y=mx+b choosea point from graph :(0,2)\when x =0 the y=b=2

y=-x+2  

Please help explanation if possible

Answers

Answer:

612 adults

361 students

Step-by-step explanation:

To solve this question, set two equations:

Let x be number of adults and y be number of students.

As there are in total 937 people, the equation would be the sum of both adults and children:

[tex]x+y=937[/tex]

[tex]x=937-y[/tex]  ...... ( 1 )

As the total sale amount is $1109, the equation would be to add up the ticket fee:

[tex]2x+0.75y=1,109[/tex]  ...... ( 2 )

Put ( 1 ) into ( 2 ):

[tex]2(937-y)+0.75y=1,109[/tex]

[tex]1874-2y+0.75y=1,109[/tex]

[tex]-1.25y=-765[/tex]

[tex]y=763/1.25[/tex]

[tex]y=612[/tex]

Put y into ( 1 ):

[tex]x=973-612[/tex]

[tex]x=361[/tex]

Therefore there are 612 adults and 361 students.

can u help me with this?​

Answers

Answer:  Yes. The sales tax is 5% which equals $4.20 for $84

Step-by-step explanation:

[tex]\dfrac{0.60}{12}=0.05\qquad \rightarrow 5\%\\\\\\\dfrac{1.20}{24}=0.05\qquad \rightarrow 5\%\\\\\\\dfrac{1.80}{36}=0.05\qquad \rightarrow 5\%\\\\\\\dfrac{2.40}{48}=0.05\qquad \rightarrow 5\%[/tex]

The sales tax rate is proportional for the values in the table.

$84 x 0.05 = $4.20

The sales tax on a purchase of $84 is $4.20

Cheryl earns (p+3) dollars per hour. In 8 hour she earns ___ dollars.

Answers

Answer:

8p + 24 dollars

Step-by-step explanation:

1 hour = (p+3)

8 hours = 8 (p+3)

=> 8p + 24

In 8 hours, Cheryl earns 8p + 24 dollars.

Answer:

[tex]\boxed{8p + 24}[/tex]

Step-by-step explanation:

Hey there!

Well if Cheryl earns p + 3 per hour and it is asking us to find the amount made in 8 hours, we can make the following,

8(p + 3)

8*p = 8p

8*3 = 24

Together,

8p + 24

Hope this helps :)

Factor each of the following perfect square trinomials. X^2+5+6 20 points

Answers

Answer:

(x + 2)(x + 3)

Step-by-step explanation:

To factor perfect square trinomials, we have to follow a certain trick, the two factor numbers must equal b (in this case 5x) when added and equal c (in this case 6) when multiplied.

Simply enough, two numbers that do this is 2 and 3.

Therefore, our factor numbers are 2 and 3, making the last eqaution (x + 2)(x + 3)

Help! Will give brainliest.

Answers

Answer:

A. You can see which shape the bases are, how many bases there are, how many faces there are, and how many edges there are.

B. The bases are ABC and DEF, and I know because they are two congruent triangles on two opposite sides of the shape.

A) the figure is a triangular prism because there are two parallel triangle bases (ABC and EDF) connected by three rectangular sides.

B) The two bases are ABC and EDF because these are the two triangular bases.

whats the factored form of 6x 2 - 8x - 8 = 0

Answers

Answer:

x = -2/3 , 2

Step-by-step explanation:

Factor 2  out of   6 x^ 2 − 8 x − 8

2 ( 3 x^ 2 − 4 x − 4 ) = 0

Factor

2 ( 3 x + 2 ) ( x − 2 ) = 0

Set  3 x + 2  equal to  0  and solve for  x

x = -2/3

Set  x − 2  equal to  0  and solve for  x

x = 2

The final solution is all the values that make 2 ( 3 x + 2 ) ( x − 2 ) = 0

x = -2/3 , 2

Hope this can help you

 

convierte los decimales a fracción:
a) 3, 1233333 =
b) 4, 3855555 =
c) 37, 22222 =
d) 16, 2929292929 =
e) 2, 33333333 =

Answers

(a) Let x = 3.12333…. Then 100x = 312.333… and 1000x = 3123.333…, so that

1000x - 100x = 3123.333… - 312.333…

==>   900x = 2811

==>   x = 2811/900 = 937/300

(b) x = 4.38555…   ==>   100x = 438.555… and 1000x = 4385.555…

==>   900x = 3947

==>   x = 3947/900

(c) x = 37.222…   ==>   10x = 372.222…

==>   9x = 335

==>   x = 335/9

(d) x = 16.292929…   ==>   100x = 1629.292929…

==>   99x = 1613

==>   x = 1613/99

(e) x = 2.333…   ==>   10x = 23.333…

==>   9x = 21

==>   x = 21/9 = 7/3


Describe the relationship between GF and CD when the two lines are perpendicular to AB. Also describe the relationship between GF and CD is not perpendicular to AB

Answers

IF GF and CD are perpendicular to AB, one can say that they will or do not intersect each other. But if CD is not perpendicular to AB, GF and CD, do intersect at one (single) point.

What is the relationship about?

In the congruent relationship above, one can say that  two lines intersect to create a linear pair of congruent angles, and thus one can say that the lines are perpendicular in nature.

Therefore, in the case above, IF GF and CD are perpendicular to AB, one can say that they will or do not intersect each other. But if CD is not perpendicular to AB, GF and CD, do intersect at one (single) point.

Learn more about perpendicular lines from

https://brainly.com/question/379998

#SPJ1

Wo commondities A and B cost 070 and D80 respe
with 26 kg of B and the mixture issold at 085 per kg: calo
The weight (in kg) of 50 contenstants at a competition is
65 66 67 66 64 66 65 63 65 68
64 62 66 64 67 65 64 66 65 67
65 67 66 64 65 64 66 65 64 65
66 65 64 65 63 63 67 65 63 64
66 64 68 65 63 65 64 67 66 64
a. construct a frequency table for the discrete data
b. calculate, correct to 2 decimal places, the
mean,
ii. standard deviation of the data​

Answers

Answer:

b) 65.08

ii) 0.75

Step-by-step explanation:

x(Kg):                  62      63     64     65     66     67     68

Frequency(f):      1          5        11       15      10     6        2

           xf:           62       315    704    975  660  402    136

b) mean(X) = ∑xf/N = 3254/50 = 65.08

c) x - X :          -3.08    -2.08    -1.08    -0.08        0.92   1.92    2.92

   (x - X)²:        9.49     4.33      1.17       0.0064    0.85   3.69    8.53

ii) Standard deviation(S.D.):  √(∑(x - X)²/N = √(28.0664)/50 = √0.5613

 ∴ S.D.= 0.75

Find the value of the variable. If your answer is not an integer, leave it in simplest radical form.

A. 7[tex]\sqrt{2}[/tex]
B. [tex]\frac{7\sqrt{3} }{2}[/tex]
C. [tex]7\sqrt{3}[/tex]
D. [tex]\frac{7\sqrt{2} }{2}[/tex]

Answers

Answer:

7 sqrt(2)/2 =x

Step-by-step explanation:

Since this is a right triangle, we can use trig functions

sin theta = opp/ hyp

sin 45 = x/7

7 sin 45 =x

7 sqrt(2)/2 =x

Answer:

[tex]\large \boxed{\mathrm{D. \ \displaystyle \frac{7\sqrt{2} }{2 }}}[/tex]

Step-by-step explanation:

The triangle is a right triangle.

We can use trigonometric functions to solve the problem.

[tex]sin \theta = opp/hyp[/tex]

The opposite side to the angle 45 degrees is x and the hypotenuse of the triangle is 7.

[tex]sin 45 = x/7[/tex]

Multiply both sides of the equation by 7.

[tex]7 sin 45 = x[/tex]

Simplify the value.

[tex]\displaystyle \frac{7\sqrt{2} }{2 }=x[/tex]

PLEASEEEEEEE HELP with this question

Answers

Answer:

second table

Step-by-step explanation:

Out of the 8 options on the spinner, 2 of them are 0's, 1 of them is a 1, 2 of them are 2's and 3 of them are 3's so the probability of spinning a 0, 1, 2 or 3 is 2/8, 1/8, 2/8 or 3/8 which becomes 0.25, 0.125, 0.25 or 0.375 respectively. Therefore, the answer is the second table.

PLEASE HELP!! Find the coordinates.

Answers

Answer:

Option (D)

Step-by-step explanation:

Coordinates of the points J, E and V are,

J → (-4, -5)

E → (-4, -3)

V → (-1, -1)

This triangle is translated by the rule [tex]T_{<2,4>}[/tex] given in the question.

Coordinates of the image will follow the rule,

(x, y) → [(x + 2), (y + 4)]

following this rule coordinates of the image triangle will be,

J(-4, 5) → J'(-2, -1)

E(-4, -3) → E'(-2, 1)

V(-1, -1) → V'(1, 3)

Therefore, points given in the option (D) will be the answer.

A
Find the value of x. Your answer must be exact.
30
9

Answers

Tan30=9/x so it will be 9((3)^1/2)

Answer:

x = 9√3

Step-by-step explanation:

Since the figure above is a right angled triangle we can use trigonometric ratios to find x

To find x we use tan

tan∅ = opposite/ adjacent

From the question

The opposite is 9

The adjacent is x

Substitute the values into the above formula

That's

[tex] \tan(30) = \frac{9}{x} [/tex]

[tex]x \tan(30) = 9[/tex]

Divide both sides by tan 30

[tex]x = \frac{9}{ \tan(30) } [/tex]

We have the final answer as

x = 9√3

Hope this helps you

Given 4,7,10,13. Determine a rule to describe the general term, Tn.​

Answers

Answer:

[tex]T_{n}[/tex] = 3n + 1

Step-by-step explanation:

There is a common difference between consecutive terms, that is

7 - 4 = 10 - 7 = 13 - 10 = 3

This indicates the sequence is arithmetic with nth term

[tex]T_{n}[/tex] = a₁ + (n - 1)d

where a₁ is the first term and d the common difference

Here a₁ = 4 and d = 3 , then

[tex]T_{n}[/tex] = 4 + 3(n - 1) = 4 + 3n - 3 = 3n + 1

Other Questions
Read Case Study 5:2 Gas or Grouse? in Chapter 5 of your textbook Business Ethics Concepts & Cases (8th Edition) by Manuel G. Velasquez and select the best answer for multiple choice questions below. 1. the spending bill was: a. not necessary for the adjudication of Gas b. passed by a largely republican congress c. one of the most important milestones for emissions in the last century d. the nickname of a politician that overspent funds by the name of William 2. BLM argued that: a. it is important to treat everybody with respect b. not everybody deserves an opportunity c. gas emissions are important inter-globally d. a significant decline in deer was a cause of action 3. The directional technology to drill was between: a.$400,000 - $600,000 b. democrats and republicans c. scientists and physicists d. the period of 2008-2013 4. Grouse could be affected by: a. folic acid in the airb. excavation for silver and coal c. overpopulation of Grouse d. things that attract ravens 5. A decline in number of birds resulted because of a. destruction of grounds for mating b. 80% of nesting was destroyedc. poor government internal controls d. necessary bird control How to solve this question pls help asap How did Shi Huangdi attempt to defend the Qin Empire from attacks by nomads?O Built the Silk RoadO Completed a 2,500 mile wallO Wrote Hammurabi's CodeO Ended legalism in the government write a conversation on my school flower in nepali The excerpt is from a memoir becauseA) it introduces fictional characters B) it reveals a futuristic narrative C)it contains a objective researched facts D) it shares the writers emotional responses What is the value of the equilibrium constant, K, for a reaction for which G is equal to 5.20 kJ at 50C? Pls help me with this question Data regarding four different products manufactured by an organization are presented below. The manufacturer has a constrained resource - machine hours.Product A Product B Product Product DSelling price per unit $20.00 $25.00 $23.00 $15.00Variable cost per unit $10.00 $16.00 $11.00 $7.00Hours to make each unit 5 hours 25 hours 2 hours 35 hours Rank these four products in order of profitability.12 34 Fast please!!. Serious answers only. Considering that Susan is 38 years old, what is the probability that she lives to the age of 85? plz translate to English and tell the language name For which one of the following reactions will the enthalpy change be approximately equal to the internal energy change? A. H2 + I2 2HI B. PCl5(g) PCl3(g) + Cl2C. 2H2O2 2H2O2 + O2D. C(s) + O2(g) CO2(g) Which of the following transitions would be used to show a sequence?O In light of thisO FinallyThereforeAs a result Most cars have a coolant reservoir to catch radiator fluid that may overflow when the engine is hot. A radiator is made of copper and is filled to its 21.1 L capacity when at 12.2C. What volume of radiator fluid (in L) will overflow when the radiator and fluid reach a temperature of 95.0C, given that the fluid's volume coefficient of expansion is = 400 106/C? (Your answer will be a conservative estimate, as most car radiators have operating temperatures greater than 95.0C). Mr. Clifton was preparing for his upcoming birthday party. He ordered 51 white balloons and 47 red balloons. As he walked to his car 12 balloons flew away! With the remaining balloons, he mixed the colors to create bunches of 3 to decorate his party space. How many full bunches did Mr. Clifton have to decorate his party space? Fill in the blanks to complete the sentence. Fixed costs equal $25,000; variable cost per unit is $2.50 and units produced are 10,000. The total budgeted costs is Figure B is a scaled copy of Figure A.93.63.69Figure A1.231.23Figure BWhat is the scale factor from Figure A to Figure B? what does this equal 2^3 + 6^5= The following data were taken from the records of Clarkson Company for the fiscal year ended June 30, 2020. Raw Materials Inventory 7/1/19 $48,100 Factory Insurance $4,700 Raw Materials Inventory 6/30/20 39,700 Factory Machinery Depreciation 16,100 Finished Goods Inventory 7/1/19 96,100 Factory Utilities 28,700 Finished Goods Inventory 6/30/20 19,900 Office Utilities Expense 8,550 Work in Process Inventory 7/1/19 19,900 Sales Revenue 555,000 Work in Process Inventory 6/30/20 19,900 Sales Discounts 4,300 Direct Labor 139,350 Plant Managers Salary 61,100 Indirect Labor 24,560 Factory Property Taxes 9,610 Accounts Receivable 27,100 Factory Repairs 1,500 Raw Materials Purchases 96,500 Cash 32,100Required:Prepare an income statement through gross profit. PLS HELP! Using the diagram shown below, which statement is true? HELP . Read the following graduated cylinder.Use this media to help you complete the question.22.0 mL22 mL23.0 mL22.5 mL